Pulmonology

Pataasin ang iyong marka sa homework at exams ngayon gamit ang Quizwiz!

A 50-year-old obese woman undergoes a cholecystectomy and T-tube drainage of the common bile duct. On postoperative day 7, she develops sudden epigastric and left-sided chest pain. She is short of breath and is sweating profusely. Her temperature has been between 99-100°F for the past 2 days. The patient's vitals are: pulse 90/min, BP 110/60 mm Hg, and RR 24/min. The ECG reading shows tachycardia, and the ventilation-perfusion lung scan shows a VQ mismatch. Question Based on the most likely diagnosis, what should be the initial management of this patient? 1 Pulmonary embolectomy 2 Urokinase therapy 3 Exploratory laparotomy 4 Heparinization 5 Inferior vena cava filter

4. Heparinization

You are currently on an inpatient hospitalist team in a local pediatric hospital. First thing this morning, your team is called in to evaluate an infant born at 27 weeks gestation 50 minutes ago. Upon initial inspection of the newborn, you observe rapid labored grunting respirations, flaring nostrils, and retractions that are present above and below the breastbone. Auscultation reveals diminished air movement, and a chest radiograph reveals a ground glass appearance in the lung fields bilaterally. Question What pharmacological agent should be initiated to help alleviate these signs and symptoms? 1 Intravenous antivirals 2 Hepatitis B immunoglobulin 3 Intravenous antibiotic 4 Inhaled surfactant replacement 5 Inhaled corticosteroids

4. Inhaled surfactant replacement

A newborn infant is delivered at 30 weeks since the last menstrual period. At birth, the infant weighs 1500 gm but otherwise appears normal. Soon after birth, the infant becomes cyanotic and breathes with a grunting noise. Chest X-rays reveal dense lungs with significant atelectasis but no cardiovascular abnormalities. Question What is the most likely diagnosis? 1 Congenital diaphragmatic hernia 2 Coarctation of the aorta 3 Tetralogy of Fallot 4 Bilateral renal agenesis 5 Respiratory distress syndrome

5. Respiratory distress syndrome

A 27-year-old male accident victim with a head injury is admitted to the ICU and kept on mechanical ventilatory support. On the seventh day after admission, he is clinically diagnosed with pneumonia. Blood samples and lower respiratory secretions are submitted to the laboratory for culture; empiric antimicrobial therapy is started. Question What is the most likely etiologic agent of pneumonia in this patient? 1 Streptococcus pneumoniae 2 Klebsiella pneumoniae 3 Mycoplasma pneumoniae 4 Moraxella catarrhalis 5 Haemophilus influenzae

2. Klebsiella pneumoniae

A 55-year-old man presents with a COPD exacerbation managed with a ventilator. The patient's blood pressure drops and the ventilator alarm goes off. The only medication being administered is amlodipine via nasogastric tube. This patient is afebrile, even though the hospital has been having problems with pseudomonas infection in ventilated patients. On examination, there is a middle-aged orally-intubated man with temperature 99.4°F, pulse 145/min, and BP 62/34 mm Hg; he breathes above the ventilator at a rate of 36 cycles/min. His breathing is shallow, and there are diminished breath sounds in the right hemithorax. Question What is the most appropriate next step? 1 Add positive end-expiratory pressure. 2 Perform needle thoracotomy, chest tube placement. 3 Start antibiotics. 4 Give IV fluid boluses. 5 Start pressor agents.

2. Perform needle thoracotomy, chest tube placement

A 76-year-old man with hypertension, coronary artery disease, and poorly-controlled left ventricular congestive heart failure presents with a 3-day history of insidious chest pain. Pain is made worse when he takes a deep breath in and when he coughs. He denies any relation of pain to position, activity, or food intake. He denies fever, chills, palpitations, sputum production, wheezing, abdominal pain, nausea, vomiting, diarrhea, or peripheral edema. His physical exam reveals a widespread friction rub upon inspiration, absent lung fremitus, and reduced lung sounds over the thoracic cavity. Question What finding would be anticipated upon further diagnostic testing? 1 Serum glucose to pleural ratio of greater than 1.0 on pleural fluid analysis 2 Pleural fluid N-terminal pro-brain natriuretic peptide levels greater than 1500 pg/mL 3 Lobar consolidations with air bronchograms on chest radiography 4 Hyperinflation, hyperlucency, and depressed diaphragms on chest X-ray 5 Pleural to serum LDH ratio of greater than 0.6

2. Pleural fluid N-terminal pro-brain natriuretic peptide levels greater than 1500 pg/mL

A 57-year-old man presents with a 6-month history of a daily productive cough. The patient is a non-smoker, and he has worked in a local coal mine for the past 39 years. He very rarely comes in to see a healthcare provider. When asked if he is up to date with his vaccinations, he does not recall the last ones he received; he also does not recall when these may have been given. Question Given the most likely diagnosis, what pair of vaccinations would be recommended to help decrease any significant morbidity and mortality in this patient? 1 Influenzae and pneumococci 2 Influenzae and zoster 3 Pneumococci and varicella 4 Pneumococci and zoster 5 Varicella and zoster

1. Influenzae and pneumococci

A 3-year-old boy presents with a 4-hour history of respiratory distress, dysphagia, and fever. On examination, temperature is 104.3°F, pulse 150, and respirations 32 and shallow. Marked inspiratory stridor with an open-mouth appearance and sialorrhea is present. Question What is the initial step in management? 1 Arterial blood gas analysis 2 Intravenous cephalosporins 3 Securing the airway 4 Soft tissue lateral neck radiograph 5 Visualizing epiglottis with tongue blade

3. Securing the airway

A 36-year-old non-smoking man is diagnosed with emphysema. Upon further questioning, he tells you his father was also diagnosed with emphysema when he was in his 30s and never smoked. He states that his father passed away at age 50 of hepatocellular carcinoma. Question What diagnostic study should the patient have done? 1 Interferon-gamma release assay 2 Serum alpha-antitrypsin (AAt) 3 Serum angiotensin-converting enzyme (ACE) 4 Serum brain natriuretic peptide (BNP) 5 Sweat chloride test

2. Serum alpha-antitrypsin

You have just taken over the management of a 55-year-old man with COPD who was admitted 3 days earlier for community-acquired pneumonia. He currently feels somewhat better, and he has been afebrile for the last 24 hours. Leukocytes count 5,600/μL Serum glucose 106 mg/dL Segmented neutrophils 75% Sodium 138 mmol/L Hemoglobin 19g/dL Chloride 102 mmol/L Platelets 245,000/μL Potassium 4.2 mmol/L Arterial blood gas PH 7.25 Bicarbonate 29 mmol/L PCO2 55 BUN 18 mmol/L PO2 57 Creatinine 1.0 mmol/L HCO3 29 O2 sat 88% Question What has been shown to improve life expectancy in a patient like this? 1 Antibiotics 2 Bronchodilator therapy 3 Home oxygen 4 Inhaled steroids 5 Acetazolamide

3. Home oxygen

A 50-year-old man presents with a 2-week history of not being able to see well. He is not on any medications. He has been smoking 2 packs of cigarettes a day for the past 30 years. On examination of his right eye, there is ptosis and miosis. A chest radiograph reveals a rounded opacity in the right lung field. Question What is the most likely diagnosis? 1 Lambert-Eaton syndrome 2 Superior vena cava syndrome 3 Horner syndrome 4 Cushing syndrome 5 Syndrome of inappropriate antidiuretic hormone secretion

3. Honer Syndrome

A 62-year-old man presents for evaluation of facial swelling that feels worse with bending forward. He states he has also experienced headaches, shortness of breath, and visual problems over the past few weeks. He admits that he has a 70 pack-year smoking history. Upon examination, you note swelling of the face and distention of neck and chest veins. You appreciate diminished breath sounds and tactile fremitus in the right upper lobe. Question What is the most likely diagnosis? 1 Interstitial lung disease 2 Spontaneous primary pneumothorax 3 Superior vena cava obstruction 4 Pneumonia 5 Chronic bronchitis

3. Superior vena cava obstruction

Routine physical examination of a 55-year-old man demonstrates marked finger clubbing. Radiography of the hand shows new bone formation beneath the periosteum. Question This finding is most strongly associated with what disorder? 1 Chronic renal failure 2 Colon cancer 3 Endocrine adenomas 4 Intrathoracic cancer 5 Profound anemia

4. Intrathoracic cancer

A 9-year-old boy presents with a productive cough for the past few days. The patient also has been having fevers of 101°F. The patient has had recurrent bouts of pneumonia. His neonatal course was complicated by meconium ileus. A sweat chloride test is positive and sputum examination reveals gram-negative rods that are oxidase-positive. The culture has a greenish tint. Question Based on the patient's underlying condition and laboratory findings, this infection is most likely caused by what organism? 1 Pseudomonas aeruginosa 2 Staphylococcus aureus 3 Streptococcus pneumoniae 4 Haemophilus influenzae 5 Listeria monocytogenes

1. Pseudomonas aeruginosa

A 56-year-old man presents with moderately severe substernal and left anterolateral chest pain. There is some exacerbation of pain on inspiration that has been increasing in severity over the last 36 hours. He works as a truck driver and has a history of heavy cigarette smoking, hypertension, and obesity. Over the past week, he has experienced swelling and discomfort in his right calf. Examination shows BP of 90/55 mm Hg, P of 122/min, RR of 40/min, and temp of 37.6°C. The patient is mildly agitated and confused. Systemic examination reveals tachycardia, soft systolic murmur, and questionable ventricular gallop. Lungs show dullness to percussion at left base, with scattered crackles and wheezes throughout. Abdominal and neurological exams are negative. The right calf is 0.5 cm larger than left, with some deep tenderness and a trace of ankle edema. Laboratory analysis reveals hemoglobin 16.4 g/dL, Hct 51%, WBC 12,300 cells/µL, PaO2 52 mm Hg, PaCO2 38 mm Hg, and pH 7.35. Chest radiograph shows borderline cardiomegaly and a prominent aorta, scattered patchy infiltrates bilaterally, and a small left pleural effusion. Question What is the most accurate diagnostic modality for diagnosing this patient's condition? 1 Cardiac ultrasound 2 Computerized tomographic angiography 3 Impedance plethysmography 4 Myocardial scan 5 Spirometry

2. Computerized tomographic angiography

A 32-year-old man presents due to occasional shortness of breath and associated cough, especially when he is working outside. He has associated chest tightness that resolves within minutes when he sits down and rests. These symptoms occur 1-2 days a month. He is otherwise healthy and does not smoke. Blood pressure is 128/74 mm Hg, and pulse is 76, respiration is 14, and pulse oximetry is 100% on room air. His FEV1 is 96%. Question What is the treatment of choice? 1 Inhaled corticosteroid 2 Inhaled short-acting beta agonist 3 Inhaled long-acting beta agonist 4 Inhaled anticholinergic 5 Oral beta agonist

2. Inhaled Short-acting beta agonist

A 3-month-old male infant presents with history of noisy breathing since birth; the noise is gradually increasing. There is no history of fever or running nose, but there is bark-like cough present. Physical examination reveals a low-pitched expiratory wheeze loudest over the trachea. Wheezing increases during crying, feeding, and when the infant is laid in supine position. There is no cyanosis, subcostal or intercostal retraction, or hoarseness of voice. Wheezing has not shown any response to bronchodilators. Question What is the most likely diagnosis? 1 Acute bronchiolitis 2 Congenital subglottic stenosis 3 Congenital tracheomalacia 4 Tracheoesophageal fistula 5 Vocal cord paralysis

3. Congenital tracheomacia

A 16-year-old girl presents with shortness of breath, wheezing in her chest, and swelling of the lips that started several minutes after she had a snack. She had similar symptoms several months ago at a Chinese buffet. She appears restless. Her face is erythematous and her lips and tongue are swollen. She is able to repeat her name when asked. Her blood pressure is 89/60 mm Hg, and heart rate is 110/min rhythm regular, with respirations 26/min and shallow. On auscultation, diffuse wheezing is heard. Question What is the next step in management of this patient? 1 Take arterial blood gases. 2 Intubate the patient. 3 Give epinephrine. 4 Give corticosteroid. 5 Give a beta blocker

3. Give epinephrine

A 52-year-old woman who lives a non-sedentary lifestyle presents with a 5-day history of low-grade fever, flu-like syndrome, sore throat, and malaise. She has to catch her breath because of pain on inspiration and when coughing. She has no known past medical or surgical history; she is not on any medication, and she has no pertinent family history. She denies any medication use, including over-the-counter medicines. On physical examination, her vitals are: temperature 100.6°F, pulse 86/min, BP 133/75 mm Hg, and RR 20 cycles/min. She has shallow breathing, resonant percussion notes, fair air entry with vesicular breath sounds, and friction rub. Her blood gas on room air is as follows: pH 7.36 PCO2 44 mm Hg PO2 100 mm Hg HCO3 26 mEq\L O2 saturation 99.8% Her chest X-ray (CXR) and D-dimer assay are normal. Question What is the most appropriate management for this patient? 1 Albuterol 2 Heparin 3 NSAIDs 4 Oseltamivir 5 Furosemide

3. NSAIDs

A 62-year-old man presents with shortness of breath, some chest discomfort, and palpitations. Upon examination, his pulse rate is 200/min; blood pressure is 100/75 mm Hg, and he has an oral temperature of 98.7°F. A 12-lead ECG examination reveals the wave patterns in the figure below (Refer to the image). He has a history of chronic bronchitis. He has been taking drugs to control his condition since he quit smoking 6 months ago. Question What treatment is most likely causing his symptoms? 1 Inhaled corticosteroids 2 Oral steroids 3 Phosphodiesterase-4-inhibitor 4 Theophylline 5 Antibiotics

4. Theophylline

A 30-year-old immunocompromised patient presents with a 2-week history of breathlessness and a nonproductive dry cough. The patient is afebrile, pulse is 100, and BP is 110/70 mm Hg. On auscultation, scattered rales all over the chest are heard. A chest X-ray shows diffuse air-space and interstitial shadowing in both lungs. The shadowing is more prevalent in the apical region. Question What is the most likely diagnosis? 1 Tuberculosis 2 Mycoplasma pneumoniae 3 Viral pneumonia 4 Streptococcus pneumoniae 5 Pneumocystis pneumoniae

5. Pneumocystis pneumoniae

A 27-year-old woman whose pregnancy was complicated by gestational diabetes went into premature labor and vaginally delivered a 33-week-old male. Minutes after delivery, the newborn became cyanotic and tachypneic and had intercostal retractions. A stat chest X-ray showed a diffuse ground glass appearance, hypoventilation, and air bronchograms. Question What physiologic abnormality accounts for this newborn's condition? 1 Decreased production of nitric oxide 2 Decreased production of surfactant 3 Decreased release of catecholamine 4 Increased release of cytokines 5 Increased resorption of sodium

2. Decreased production of surfactant

A 24-year-old man undergoes a routine medical check-up to become a volunteer in the ER. PPD skin test shows an induration diameter of 9 mm. Sputum and chest X-ray are done; they are negative for tuberculosis. He is otherwise healthy and has no fever, cough, or other symptoms of Mycobacterium tuberculosis infection. Question What is the most appropriate explanation for the doubtful tuberculin test in this patient? 1 Sputum and chest X-ray report could be wrong. 2 Adequate sputum samples were not supplied. 3 He was given BCG vaccination earlier in life. 4 He may suffer from chronic cavitary lesion in the lung. 5 He has an active tubercular infection.

3. He was given BCG vaccination earlier in life.

A 6-year-old boy with Down syndrome is brought to the emergency department after a drowning accident in a lake. Submersion time is unknown, but he was missing for some time. He was resuscitated and intubated by emergency medical technicians during transport. On arrival, his heart rate is 76 beats/min, BP 104/72 mm Hg, and rectal temperature is 81°F (27°C). He remains comatose, exhibiting non-purposeful flexion withdrawal to pain. His pupils are equal and reactive. Question What is the best way to treat this patient's drowning-related hypothermia? 1 Passive rewarming 2 Active external warming 3 Active internal warming 4 Maintain this degree of hypothermia 5 No warming measure necessary

3. Active internal warming

A 60-year-old woman presents with a history of persistent cough. She is confined to her bed; walking over 10 paces causes severe breathlessness. She has no energy to carry out any of her regular activities. She has never smoked, and she drinks the occasional glass of wine. On physical examination, she is found to have decreased breath sounds and dullness to percussion over her right lower thorax. Further evaluation reveals an irregular mass in the periphery of the right lung base with a right-sided pleural effusion. A needle is inserted into the pleural space and divulges bloodstained fluid. Question If results prove to be a malignancy, what is the most likely pathology of her mass? 1 Small cell carcinoma 2 Large cell carcinoma 3 Adenocarcinoma 4 Mesothelioma 5 Squamous cell carcinoma

3. Adenocarcinoma

A mother brings her 8-month-old son to your office in January. She reports he has been fussy and has had some difficulty feeding over the last few days. Today, his symptoms have worsened. On exam, the infant is noted to be short of breath, wheezing, with intercostal retractions, and respiratory rate of 50. Question What exam finding would further suggest your suspected diagnosis? 1 Wheezing improved after nebulizer 2 Peak flow 300 prior to treatment 3 Liver and spleen palpable 4 Clubbing of the finger 5 Diffuse maculopapular rash

3. Liver and spleen palpable

A 41-year-old woman presents due to worsening symptoms. She was diagnosed with idiopathic pulmonary hypertension about 2 years ago; she is on home oxygen therapy. She has longstanding fatigue and dyspnea, but she is now experiencing profound dyspnea with exertion, swelling in her ankles, some discomfort in her right upper abdomen, and the inability to breathe well when lying down. She has always been thin, but her weight has increased by 10 pounds in the last month. She denies fever and chills. She recently had an electrocardiogram (ECG), but she has not seen a healthcare provider to discuss the results. The ECG report indicates peaked P waves, right axis deviation, and tall R wave in V1. Question Based on this patient's history and test results, what physical exam findings would be expected? 1 Abdominal bruit 2 Absent breath sounds in right lung 3 Dry mucous membranes and reduced skin turgor 4 Hepatojugular reflux 5 Virchow's node enlargement

4. Hepatojugular reflux

A 66-year-old non-smoking man presents to the family practice clinic due to chronic cough. He notes shortness of breath on exertion. On physical exam, an increased respiratory rate with shallow breathing is noted. Dry crackles are auscultated bilaterally over the lungs. No clubbing or cyanosis is noted. The remainder of the exam and vitals are normal. This patient has the following test results: Pulse oximetry Slightly hypoxic Chest X-ray Small opacities Pulmonary function tests (PFTs) Restrictive pattern Question In evaluation of this patient's chronic cough, what aspect of his history is most helpful in distinguishing idiopathic pulmonary fibrosis from the various types of pneumoconioses? 1 Character of cough 2 Duration of cough 3 Family history of pulmonary disease 4 Occupational exposure history 5 Past surgical history

4. Occupational exposure history

A 75-year-old man is recovering from a pneumonia caused by Streptococcus pneumoniae; his condition suddenly deteriorates. He presents after developing a persistent fever, chills, cough, and diaphoresis. A CBC reveals leukocytosis with a left-shift. A chest X-ray demonstrates an air-fluid level in the pleural space, which is suggestive of an abscess. Question What organism caused the patient's deterioration? 1 Pseudomonas aeruginosa 2 Chlamydia trachomatis 3 Coccidioides immitis 4 Staphylococcus aureus 5 Legionella pneumophila

4. Staphylococcus aureus

A family presents in the middle of winter. They live in a low-income housing development. Due to their gas furnace being broken, they have been using a kerosene heater at night for the past week. For the last 3 days, both parents and the two siblings have been experiencing varying degrees of headache, dizziness, nausea, vomiting, and fatigue; symptoms are particularly severe at night and have been increasing in intensity each night they occur. The 4-year-old daughter has been very lethargic; occasionally, she seems to black out or fall asleep very soundly. She and her 9-year-old brother have also had a cough, runny nose, and sore throat for the past week. On exam, other than seeming tired, findings are nonspecific. Both children have a runny nose but their lungs and ears are clear. Question What is the best test to confirm exposure to the most likely diagnosis in this case? 1 Hemoglobin level 2 Pulse oximetry 3 Blood gas analysis 4 Urinalysis 5 Carboxyhemoglobin level

5. Carboxyhemoglobin level

A 57-year-old man presents with progressive dyspnea on exertion and left lumbar colic. He has a history of hypertension as well as a 40 pack-year history of smoking. He denies cough, orthopnea, and paroxysmal nocturnal dyspnea. He has some mild ankle swelling, but he has no history of congestive heart failure. The only medication he is on is amlodipine. His vital signs are as follows: temperature 99.8°F, pulse 92/min, respiration 22/min, BP 128/88 mm Hg. Of significance on physical examination is the absence of breath sounds in the left lower lung zone. Laboratory data reveals WBC 1000/μL with 70% segmented neutrophils, serum glucose 106 mg/dL, sodium 138 mmol/L, chloride 102 mmol/L, potassium 4.2 mmol/L, bicarbonate 22 mmol/L, BUN 32 mmol/L, creatinine 1.2 mmol/L, protein 8.2 g/dL, amylase 56 U/dL, and LDH 250 U/mL. Thoracentesis is done and pleural fluid analysis shows WBC 910/μL, RBC 14/μL, LDH 108 U/mL, protein 2.6 g/dL, glucose 82 mg/dL, and creatinine 1.2 mmol/L. Question What is a possible cause of this condition? 1 Rheumatoid arthritis 2 Pancreatitis 3 Pulmonary embolism 4 Bacterial pneumonia 5 Cirrhosis

5. Cirrhosis

A 67-year-old man presents with dyspnea and chest pain 3 days after flying home from Europe. Workup reveals a pulmonary embolism. The patient has required 6 L of O2 to maintain a saturation above 90% and has continued to remain tachycardic. The remainder of his past medical history is unremarkable. Question What therapy should you initiate at this time? 1 Aspirin plus rivaroxaban (Xarelto) 2 Warfarin (Coumadin) plus clopidogrel (Plavix) 3 Clopidogrel (Plavix) plus apixaban (Eliquis) 4 Enoxaparin sodium (Lovenox) plus aspirin 5 Enoxaparin sodium (Lovenox) plus warfarin (Coumadin)

5. Enoxaparin sodium (Lovenox) plus warfarin (Coumadin)

A 64-year-old man presents with progressive dyspnea, fatigue, chronic dry cough, and exercise intolerance. His symptoms have been progressing over the past several months to a year. As part of your workup, you obtain pulmonary function testing; it reveals an FEV1/FVC ratio of >0.7, a decreased total lung capacity, and a decreased residual volume. Question What is the most likely diagnosis? 1 Anemia 2 Asthma 3 Chronic obstructive pulmonary disease 4 Pulmonary embolism 5 Idiopathic pulmonary fibrosis

5. Idiopathic pulmonary fibrosis

A 45-year-old woman presents with insidious onset of increasing dyspnea. A chest X-ray revealed nodular infiltrates and marked hilar lymphadenopathy. The transbronchial biopsy demonstrated non-necrotizing granulomas. Question What is the most likely diagnosis? 1 Asbestosis 2 Idiopathic pulmonary fibrosis 3 Coccidioidomycosis 4 Hypersensitivity pneumonitis 5 Sarcoidosis

5. Sarcoidosis

A 35-year-old woman presents with 5-hour history of progressive shortness of breath, cough, and wheezing. This morning she felt that she was "catching a cold" because of sore throat and thin purulent rhinorrhea, for which she took aspirin. Her past medical history is significant for persistent rhinitis resistant to therapy. Question What should your patient do to prevent future asthma attacks? 1 Avoid aspirin. 2 Take antihistamine. 3 Inhale cromolyn. 4 Take albuterol. 5 Get influenza vaccine.

1. Avoid aspirin

A 36-year-old African American woman presents with a nonproductive cough, malaise, mild fever, and mild dyspnea. She also indicates that she has some lesions around her nose. Upon physical exam, you note red-brown dermal papules around her nares. A chest X-ray demonstrates a right hilar mass. A pulmonologist is consulted and performs a biopsy during bronchoscopy. The report reveals that the mass is a non-caseating granuloma. Question What initial therapy should be prescribed? 1 Corticosteroids 2 Inhaled beta agonists 3 Methotrexate 4 Amphotericin B 5 Cyclosporine

1. Corticosteroids

A 4-year-old girl has always been below 3% for her height and weight. Her mother says she eats three meals a day with healthy snacks. The family tries to maintain a low-fat high-fiber diet. Her parents and sisters are of above average height and average weight. Other than occasional upper respiratory infections and one episode of pneumonia last winter, the girl has been fairly healthy. Her failure to thrive workup is normal, other than a sweat test, which reveals a high chloride concentration. Question What is an important modification to her diet as a result of the most likely diagnosis? 1 Fat-soluble vitamin supplements 2 Low-protein foods 3 Low-fat foods 4 High-calcium foods 5 Water-soluble vitamin supplements

1. Fat-soluble vitamin supplements

A 54-year-old man with emphysema presents with a blood pressure of 157/101 mm Hg. Over the next several months, he is prescribed angiotensin-converting enzyme inhibitors, diuretics, and calcium channel blockers, but the patient has to discontinue each agent because of undesirable side effects. He is prescribed a beta antagonist instead. Question What beta antagonist would be most appropriate for this patient? 1 Metoprolol 2 Nadolol 3 Propranolol 4 Sotalol 5 Timolol

1. Metoprolol

A 66-year-old man presents with vomiting. He has lung cancer, but he has declined treatment. His father died of lung cancer at the age of 65. On examination, he is confused and dehydrated with a BP of 100/50 mm Hg and HR of 100 beats/minute. Laboratory analysis reveals a calcium level of 16 mg/dL. His renal function tests are normal. Question What is the most appropriate initial therapy in the management of this patient? 1 Normal saline 2 Prednisone 3 Denosumab 4 Hemodialysis 5 Plicamycin

1. Normal Saline

A 52-year-old man has a past medical history of smoking 40 packs of cigarettes per year; he presents for a follow-up. He notes that, over the past week, he has developed increased nonproductive cough and shortness of breath. He has had a chronically-progressive cough and shortness of breath upon exertion over the past year. His occupational history reveals coal mining for 25 years. He denies any alcohol consumption, travel history, or sick contacts. He also denies fever, chills, sore throat, otalgia, chest or abdominal pain, peripheral edema, rashes, and pruritus. A chest radiograph is performed, which reveals diffuse bilateral ground-glass opacities and eggshell calcification of hilar lymph nodes. Question Based on the most likely diagnosis, what preventive medicine factor is true? 1 Smoking cessation strategies are encouraged. 2 Influenza and pneumococcal pneumonia immunizations are contraindicated. 3 A 15-mm induration upon PPD testing indicates a positive test result. 4 Low-level occupational exposure may be safely tolerated at this time. 5 Dietary and activity restrictions are necessary in disease management.

1. Smoking cessation strategies are encouraged.

A 66-year-old man with moderately well-controlled type 2 diabetes mellitus presents to his primary care provider for the management of pneumonia. His influenza test was negative. He is currently being treated with levofloxacin. He is afebrile; pulse is 93, blood pressure is 130/90 mm Hg, respirations are 18/min, and oxygen saturation is 92% on room air. Question What is the most common bacterial cause of this patient's pneumonia? 1 Streptococcus pneumoniae 2 Haemophilus influenzae 3 Staphylococcus aureus 4 Pseudomonas aeruginosa 5 Moraxella catarrhalis

1. Streptococcus pneumoniae

A 3-year-old Caucasian girl presents with her parents for followup after her third episode of pneumonia this year. Her parents report she has been acting more like herself and appears to be feeling better. On exam, she is afebrile and breathing comfortably. She has moderate crackles in the lower right lung base. Past medical history is significant for a few episodes of pneumonia each winter since birth. She has always been small for her age, but her mother says she has a healthy appetite. Her parents and brother are of medium stature. She takes no medication other than the antibiotic that was prescribed 5 days ago. Question What test should be ordered next? 1 Sweat chloride test 2 DNA analysis 3 Pulmonary function tests 4 Abdominal ultrasound 5 CT of the chest

1. Sweat chloride test

A 72-year-old man presents with progressive shortness of breath over the years. He denies chest pain or a history of smoking. The patient was in the construction business for many years, and before that he worked as a shipbuilder. Chest X-ray reveals marked interstitial fibrosis and calcified pleural plaques on the lateral chest wall. Question What is the most likely diagnosis? 1 Silicosis 2 Asbestosis 3 Siderosis 4 Byssinosis 5 Tuberculosis

2. Asbestosis

A term newborn is apneic at birth. After providing warmth and positioning and clearing the airway, the infant is still apneic and has central cyanosis; heart rate is 80 beats per minute. Question What is the next appropriate step for resuscitation? 1 Stimulate by flicking the soles. 2 Begin positive pressure ventilation. 3 Begin chest compressions. 4 Administer epinephrine. 5 Begin positive pressure ventilation and chest compressions.

2. Begin positive pressure ventilation

A 50-year-old man presents with a 2-week history of not being able to see well. He is not on any medications. He has a 60 pack-year history of cigarette smoking. Past medical and surgical history are otherwise unremarkable; he does not take any medications. On examination of his right eye, both ptosis and miosis are noted. A chest X-ray shows a rounded opacity in the right lung field. Question What is the most likely diagnosis? 1 Cushing syndrome 2 Horner syndrome 3 Lambert-Eaton syndrome 4 Superior vena cava syndrome 5 Syndrome of inappropriate antidiuretic hormone secretion

2. Horner Syndrome

A 2-week-old female neonate has been reported to have an elevated immunoreactive trypsinogen (IRT) level on her newborn screening sample that was sent from the newborn nursery after birth. The state newborn screening lab ran additional tests to screen for the 40 most common mutations known to cause the suspected diagnosis. The result of that mutation screening was negative. Question What test will definitely rule out the diagnosis? 1 Nasal epithelial potential difference measurement 2 Sweat chloride test 3 Sweat conductivity testing 4 Fecal elastase level 5 Throat swab

2. Sweat chloride test

A 44-year-old male salesperson presents after driving cross-country in a rural part of southern Ohio. His car had broken down and he found a local farmer who let the salesman spend the night in his farmhouse. His car was repaired the next day, and he promptly returned home. Several weeks later, he experienced an abrupt onset of a fever, non-productive cough, shortness of breath, and mild chest pain. Believing he has just caught a cold, the salesman thinks nothing of it. A few days after the onset of symptoms, he is much better, but he presents to your office for an opinion. You order a CXR that reveals airspace consolidations. Question What is the most likely cause of his symptoms? 1 Spontaneous pneumothorax 2 Goodpasture syndrome 3 Blastomycosis 4 Community-acquired pneumonia 5 Mycobacterium infection

3. Blastomycosis

A 28-year-old man with a history of crack cocaine abuse is rushed in to the emergency room. His mother found him in his apartment; he was cyanotic and severely short of breath. No other history is available. Examination reveals a young man in severe respiratory distress with temperature 99.2°F, pulse 102/min, respiration 40/min, BP 165/95 mm Hg, and pulse oximetry of 66%. He was intubated at his apartment by EMS personnel because of lack of improvement of his pulse oximetry on 100% non-rebreathing mask. His chest X-ray shows bilateral alveolar and interstitial infiltrates. The arterial blood gas on the mechanical ventilation (set at a rate of 12 cycles/min, tidal volume 500 mL) shows the following: PAO2/Fi02 ratio = 100 mm Hg, pH 7.52, PCO2 30, PO2 55, and O2 saturation 88%. Question What can be done to improve his oxygenation? 1 Increase tidal volume. 2 Increase respiratory rate. 3 Add positive end-expiratory pressure. 4 Add positive end-inspiratory pressure. 5 Perform emergent thoracentesis

3. Add positive end-expiratory pressure

A newborn boy is evaluated in the delivery room at 1 minute of life after an emergency cesarean section due to late decelerations. He is limp, pale, and unresponsive; he has a heart rate of 78 and a slow irregular respiratory rate. What is his Apgar score? 1. 10 2. 6 3. 5 4. 2 5. 0

4. 2

A 10-year-old girl presents with recurrent bronchitis. Her past history is significant for polypectomy, nasal polyps, failure to thrive, and repeated attacks of bronchitis. Examination reveals clubbing and diffuse rhonchi on respiratory auscultation. Investigations reveal subnormal lung function, decreased exercise tolerance, and a sweat chloride concentration of 62 mEq/L (normal: <40 mEq/L). Question What is the most likely diagnosis? 1 Pulmonary aspergilloma 2 Bronchial asthma 3 Pulmonary tuberculosis 4 Cystic fibrosis 5 Primary ciliary dyskinesia

4. Cystic fibrosis

A 3-year-old boy presents with a loud cough. He developed a runny nose and irritability 2 days ago; yesterday, he began to cough loudly and felt warm. His mother tells you that the cough sounded like a wounded animal or a dog barking. The child is diagnosed with croup, and humidification is prescribed. Question What organism is most likely responsible for the patient's croup? 1 Group A streptococci 2 Haemophilus influenzae type b 3 Influenza virus 4 Parainfluenza virus 5 Rubella virus

4. Parainfluenza virus

A patient recovering from hip surgery begins to ambulate for the first time about 2 hours postoperatively. Suddenly, they experience shortness of breath. The patient becomes tachypneic and tachycardic and experiences pain on inspiration. Prior to getting out of bed, their postoperative course was unremarkable. There is no swelling; there is no palpable thrill at the incision site. Question What is your initial diagnosis? 1 Postoperative pneumonia 2 Atelectasis 3 Arteriovenous fistula at surgical site 4 Pulmonary embolism 5 Abdominal aortic aneurysm rupture

4. Pulmonary embolism

A 4-year-old boy accompanied by his mother presents with fever, sore throat, muffled voice, and breathing and swallowing difficulty. The child is leaning forward with his head and nose tilted upward and forward. He is irritable, with moderate respiratory distress and inspiratory stridor. Pulse is 94/min, BP is 110/70 mm Hg, temperature is 101°F. Question What is the next step to confirm the diagnosis? 1 Direct fiberoptic laryngoscopy in operating room 2 Indirect laryngoscopy 3 Examination with tongue depressor 4 Lateral neck radiograph 5 Complete blood count and blood culture

1. Direct fiberoptic laryngoscopy in operating room

A 63-year-old man who is 2 days status post left total hip replacement begins to report chest pain. The pain is worse with deep inspiration, and it is associated with dyspnea. His heart rate and respiratory rate are both elevated. EKG is significant for sinus tachycardia. What method is the gold standard for diagnosing the patient's suspected condition? 1 Echocardiogram 2 Chest X-ray 3 Chest CT 4 Bronchoscopy 5 Pulmonary angiography

5. Pulmonary angiography

A 3-day-old male neonate is seen in the nursery due to failure to pass meconium and two episodes of vomiting. Prenatal and perinatal histories are unremarkable. Family history reveals a brother and sister with severe asthma. Examination reveals a moderately distended abdomen without signs of tenderness. A barium enema reveals meconium ileus with distal narrowing and proximal dilatation of the colon. Meconium is passed during the procedure, relieving the distention. Question What should be ordered? 1 Chest radiograph, CT, MRI 2 Chromosome analysis of peripheral blood lymphocytes 3 Qualitative stool fat, colonoscopy, ova, parasites 4 Rectal manometry, rectal biopsy, sweat chloride 5 Urine electrolytes, CBC, TSH

4. Rectal manometry, rectal biopsy, sweat chloride

A 5-year-old girl has paroxysms of cough that increase in severity and duration. Some coughing episodes are followed by a high-pitched inspiratory noise, and vomiting has also occurred after paroxysms. Question What laboratory test could lead to the earliest confirmation of the likely diagnosis? 1 Chest X-ray 2 Blood cell analysis 3 Culture 4 Immunofluorescent antibody staining 5 PCR assay and antigen detection

5. PCR assay and antigen detection

A 45-year-old man presents with progressive dyspnea and chest discomfort. He is 2 days status post total hip arthroplasty, which was uncomplicated. He denies any trauma to the chest. On exam, his pulse is 110 bpm; respirations are 40/min. The chest radiograph reveals a wedge-shaped opacity. Lab studies reveal a higher D-dimer level. Question What is the most likely cause of his chest pain? 1 Esophageal reflux 2 Pneumothorax 3 Aortic dissection 4 Pulmonary embolism 5 Unstable angina

4. Pulmonary embolism

A 58-year-old man presents to the emergency department for a 4-hour history of chest pain and shortness of breath. He is a long-haul truck driver and noted the symptoms started while he was driving. He admits a mild cough with some blood in his sputum. The chest pain seems to be associated with breathing and gets worse with deeper inspiration. He denies fever or chills. He is a smoker. He reports no known medical conditions, no medication use, and no prior surgeries. On physical exam, the patient is mildly obese, tachypneic (respiratory rate of 22), and tachycardic (pulse of 112). He appears to be in mild distress. Lungs are normal to auscultation and percussion. Heart exam is normal. His left lower leg has some dependent edema and tenderness. The patient thinks he strained a muscle or bumped it, but he had been too worried about his shortness of breath and failed to mention this in his history. The remainder of his exam is normal. He was immediately placed on oxygen at arrival. Several tests results are available. Test Result Oxygen saturation 92% on oxygen Complete blood count Normal Comprehensive metabolic panel Normal Electrocardiogram (ECG) Normal Troponin and CK-MB Normal Chest X-ray Normal D-dimer Elevated Question Assuming all modalities are available to this patient, what intervention is most appropriate for his current condition? 1 Anticoagulation 2 Chest tube decompression 3 Hyperbaric oxygen therapy 4 Inferior vena cava filter 5 Percutaneous coronary intervention (PCI)

1. Anticoagulation

A 29-year-old man presents with a chronic respiratory infection; he is seeking the advice of an ear, nose, and throat specialist. He reports a history of recurrent respiratory infections. A biopsy of his respiratory epithelium reveals an alteration in certain epithelial structures. Question What is most likely to be abnormal? 1 Cilia 2 Desmosomes 3 Hemidesmosomes 4 Microvilli 5 Stereocilia

1. Cilia

A 62-year-old man presents with a 2-month history of worsening fatigue and shortness of breath. He has a past medical history of emphysema attributable to his 85 pack-year smoking history. The patient complains of nearly passing out while climbing the stairs in his house. He tells you that he feels like his heart races. He reports chronic shortness of breath and cough, but he now he feels like his dyspnea is dramatically worse; he can no longer sleep in his bed. He has been trying to sleep propped up in a chair at night. He is also experiencing fatigue. He has gained about 15 pounds, and he notes that he can no longer lace up his shoes. He denies fever, chills, and chest pain. His cough produces some mucus, but no hemoptysis. His vitals are shown in the table. Weight 212 lb Height 69" Body mass index 31.3 Pulse 108 Blood pressure 140/88 Temperature 98.2°F Pulse oximetry 88% On physical exam, you see a man in mild respiratory distress; he is sitting upright and leaning forward, and he uses accessory respiratory muscles for breathing. The exam is significant for reduced air movement and mild rales bilaterally in the lungs; distended neck veins; mild tachycardia with prominent P2; lower extremity edema; and right upper quadrant abdominal tenderness with hepatomegaly. Question Based upon this patient's history and physical, what is the most likely diagnosis? 1 Cor pulmonale 2 Hepatitis 3 Lung abscess 4 Pneumonia 5 Pulmonary embolism

1. Cor pulmonale

A 50-year-old man presents with a 4-day history of increasing exertional dyspnea. He has had a chronic cough for the past 3 years and attributes it to cigarette smoking. The cough had been productive of watery sputum, but it has changed to a yellowish color over the past week. He has no known allergies and reports no family history of asthma. On general appearance, he is wheezing. His temperature is 101°F, P 105/min, BP 136/86 mm Hg, and RR 30/min. Respiratory system examination reveals decreased chest wall excursion. Auscultation reveals a prolonged expiratory phase with crepitations and generalized rhonchi. Chest X-ray reveals irregular bronchovascular markings. Laboratory results reveal Hb 15 g/dL, WBC 12,000/uL, and platelets 300 x 109/L. Question What group of pathogens is most commonly associated with acute exacerbation of the patient's most likely diagnosis? 1 Haemophilus influenzae, Streptococcus pneumoniae, Moraxella catarrhalis 2 Pseudomonas aeruginosa, Moraxella catarrhalis, Chlamydia pneumoniae 3 Legionella pneumophila, Chlamydia pneumoniae, Mycoplasma pneumoniae 4 Mycoplasma pneumoniae, Legionella pneumophila, Pseudomonas aeruginosa 5 Legionella pneumophila, Streptococcus pneumoniae, Haemophilus influenzae

1. Haemophilus influenzae, Streptococcus pneumoniae, Moraxella catarrhalis

A 50-year-old woman presents with right-sided pleural effusion. Thoracentesis shows the presence of exudative serosanguineous pleural fluid and positive cytology. Question This finding is most typical for what condition? 1 Metastatic infiltrating ductal carcinoma 2 Cor pulmonale 3 Systemic lupus erythematosus 4 Staphylococcus aureus septicemia 5 Pulmonary infarction

1. Metastatic infiltrating ductal carcinoma

You are performing a physical examination on a patient with longstanding COPD. Significant findings include wheezing respirations, cyanosis, and distended neck veins; a left parasternal lift and a tender liver are both noted on palpation. Question What is most likely causing the patient's symptoms? 1 Constrictive pericarditis 2 Cor pulmonale 3 Rheumatic fever 4 Dilated cardiomyopathy 5 Congestive heart failure

2. Cor pulmonale

You are examining a 6-month-old male infant who has not received any immunizations since birth. He was born at home and has a history of multiple respiratory tract infections and chronic diarrhea since birth. His parents are not related. Family history is significant for two of his older brothers dying of pneumonia at ages 8 and 1. His 4-year-old sister is healthy. His parents moved to the US from an underdeveloped country 3 months ago. They are worried because one of their neighbors' children (age 6) developed a pruritic skin rash and fever 2 days ago; another child (age 6 months) is in the hospital because of severe vomiting and diarrhea. On examination, your patient's vitals are normal. His weight is <5th percentile, his height is at the 5th percentile, and his head circumference is at the 50th percentile. Physical exam shows an eczematous skin rash, and mucocutaneous changes suggestive to a fungal infection are noted. His thyroid, heart, lungs, abdomen, genitalia, and nervous system are within normal limits. There are no palpable lymph nodes and no hepatosplenomegaly. Question What vaccine should you recommend? 1 Rotavirus vaccine (RV) 2 Pneumococcal vaccine (PCV13) 3 Measles, mumps, and rubella vaccine (MMR) 4 Bacille Calmette-Guerin vaccine (BCG) 5 Varicella vaccine (Var)

2. Pneumococcal vaccine

A 22-year-old woman at 24 weeks gestation presents with a 3-day history of a nonproductive cough and fever. She states she has been battling an upper respiratory infection that does not seem to go away. Initially, she thought that she had a cold, but the symptoms persisted. She was told by her obstetrician that she has the flu and to drink plenty of fluids and get some rest. Vital signs reveal temperature 101.2°F, blood pressure 120/80 mm Hg, heart rate 110/bpm, and respiratory rate 22 breaths/min. On physical examination, she appears ill; lungs exhibit wheezing. Question What would be an appropriate treatment for this patient? 1 Doxycycline 2 Ciprofloxacin 3 Erythromycin 4 Amoxicillin 5 Penicillin

3. Erythromycin

A 75-year-old man presents with a 4-month history of dyspnea on exertion and a productive cough. He also unintentionally lost 10 lb in 2 months. His past medical history is significant for coronary artery disease and a myocardial infarction. He has smoked the occasional cigar over the last few years. He has been retired for 12 years, but he worked odd jobs in construction for 30 years. He also helped his father in the family's auto shop. Vital signs are normal. His physical exam is remarkable for decreased breath sounds in left lower lung fields and dullness to percussion. A chest radiograph shows a left-sided pleural effusion. Question What is the most likely diagnosis? 1 Lung cancer 2 Congestive heart failure 3 Malignant mesothelioma 4 Pneumonia 5 Recurrent postmyocardial infarction pericarditis

3. Malignant mesothelioma

A 19-year-old Caucasian man has come to see you as the last patient of the day. He presents with sudden onset of severe shortness of breath. He states that he has been an avid basketball player all his life and was practicing about 4 hours prior to his visit when he experienced sudden chest pain and immediate shortness of breath that is still bothering him currently. He describes the chest pain in the middle of the chest, more so on the right anterior side. The patient admits to smoking half a pack of cigarettes daily. Physical examination reveals a tall, thin, well-developed man in mild distress. The only other abnormalities discovered are mild tachycardia (120 beats per minute) and diminished breath sounds in the posterior right lower lobe. Question Based upon the examination so far, what is the most likely diagnosis? 1 Pneumonia 2 Pulmonary embolism 3 Pneumothorax 4 Myocardial infarction 5 Asthma

3. Pneumothorax

A 33-year-old man presents with shortness of breath, wheezing, mild fever, and fatigue. He has had several similar episodes in the past, and each previous episode began after a cold that moved into his chest. Over the past several weeks, he has had a productive cough most mornings. He has no other symptoms or exam findings. He smokes on a social basis. His CXR is normal and most recent pulmonary function tests reveal a reversible airflow limitation. Question What is the most likely diagnosis? 1 Chronic emphysema 2 Chronic bronchitis 3 Cor pulmonale 4 Asthmatic bronchitis 5 Bronchiectasis

4. Asthmatic bronchitis

A 33-year-old African American woman with no significant past medical history, who is in her 38th week of a normal pregnancy, presents with a 1-hour history of shortness of breath. She does not recall any precipitating activities or events that may have provoked these symptoms and recalls that she was laying in her bed for several hours since waking when the symptoms developed. She admits to an associated sharp, non-radiating pleuritic chest pain, as well as lower extremity swelling, which she states has been "persistent throughout the course of her pregnancy." She denies palpitations, chest pressure, cough, sputum, fever, chills, changes in weight, rashes, diaphoresis, abdominal pain, nausea, or a history of allergies. Her physical exam is noteworthy for tachypnea and tachycardia, but the rest of the vital signs are normal. Her lungs are clear to auscultation bilaterally, without wheezing, rhonchi, or crackles. Her lower extremities are remarkable for 2+ pitting edema up to the level of her knees; there is no calf tenderness, venous cords, or Homan's sign appreciated. Her skin and mucous membranes were without diaphoresis or cyanosis. A bedside EKG reveals sinus tachycardia at 120 bpm with prominent S waves in lead I and Q waves in lead III. Question What is the most likely diagnosis? 1 Acute myocardial infarction 2 Bacterial pneumonia 3 Pulmonary edema 4 Pulmonary embolism 5 Bronchial asthma

4. Pulmonary embolism

A 1-month-old premature male infant with bronchopulmonary dysplasia (BPD) remains intubated and monitored in the NICU. He has been doing relatively well and is being gradually weaned from the respirator. Suddenly, his O2 saturations and heart rate plummet, and he becomes very dusky. On quick exam, there are decreased breath sounds on the right with an asymmetric chest rise. Question What is the most likely explanation for his sudden respiratory and clinical change? 1 Inadequate tidal volume 2 Large leak around the tracheal tube 3 Displacement of the tracheal tube 4 Tension pneumothorax 5 Disconnected oxygen supply

4. Tension pneumothorax

A 4-year-old child presents in cardiopulmonary arrest after being found at the bottom of a swimming pool. CPR is administered by paramedics, and an intraosseous needle and endotracheal tube are inserted. What confirms that an intraosseous needle is correctly inserted into the marrow cavity? 1 An X-ray showing the needle is in the bone 2 A sudden increase in resistance as the needle passes through the bony cortex and reaches the other side 3 Pulsations of blood can be seen in the hub 4 The needle remains upright without support 5 Insertion of the needle up to the hub

4. The needle remains upright without support

A 67-year-old male post office worker presents with a 3-week history of increasing shortness of breath; it occurs even while he is at rest. The patient was diagnosed with congestive heart failure in the past year and he has been well controlled on oral medication. He has no history of tobacco use. He has gained 10 pounds since his last exam 2 months prior to presentation. On physical exam, there are diminished breath sounds and decreased tactile fremitus bilaterally at the base of the lungs. Dullness to percussion is also noted in the same area. He has 3+ bilateral pitting lower extremity edema. Question What is the most likely diagnosis? 1 Lung malignancy 2 Tuberculosis 3 Empyema 4 Spontaneous pneumothorax 5 Pleural effusion

5. Pleural effusion

A 69-year-old man presents with shortness of breath associated with a non-productive cough. Symptoms have progressed over the last 2 weeks. He has had a difficult time sleeping at night due to dyspnea, and his wife states he has been sleeping in a recliner. He notes stabbing chest pain that increases with inspiration. He denies fever, chills, or hemoptysis. He denies previous similar episodes. On physical exam, blood pressure 166/94 mm Hg, oxygen saturation 84% on room air, pulse 80 bpm and regular, temperature 98.8°F. Cardiac exam reveals normal S1 and S2 with S3 gallop. Jugular venous distention noted. Pulmonary exam reveals dullness to percussion, decreased tactile fremitus, and decreased breath sounds over lower left lung. He has 2+ edema to mid-knee level. A chest X-ray is ordered, which confirms a large left-sided pleural effusion. Question What is the next step in managing this patient? 1 Tube thoracostomy 2 IV furosemide 3 Oral levofloxacin 4 Pericardiocentesis 5 US-guided thoracentesis

5. US - guided thoracentesis

A 40-year-old woman undergoing chemotherapy develops fevers and respiratory symptoms. A chest radiograph is ordered; it reveals bilateral fluffy pulmonary infiltrates. A bronchoscopy with biopsy is performed; the specimen is found to contain septate hyphae with acute-single branching. Question What treatment should the patient receive? 1 Erythromycin 2 Tetracycline 3 Penicillin 4 Vancomycin 5 Voriconazole

5. Voriconazole

A 4-year-old boy accompanied by his mother presents with fever, sore throat, muffled voice, and breathing and swallowing difficulty. The child is leaning forward with his head and nose tilted upward and forward. He is irritable, with moderate respiratory distress and inspiratory stridor. Vitals are as follows: pulse is 94/min; BP is 110/70 mm Hg; temperature is 101°F. Direct fiberoptic laryngoscopy is performed and shows an edematous larynx. Cultures are taken, and an endotracheal tube is placed. The epiglottis cultures reveal Haemophilus influenzae, and the diagnosis of acute Haemophilus influenzae epiglottitis is made. The mother is worried about her 1-year-old child living in the same house and is currently not vaccinated for H. influenzae. Question What prophylactic measure would you recommend at this time? 1 All family members, including the patient, should receive prophylactic rifampin. 2 All family members, excluding the patient, should receive prophylactic rifampin. 3 Only the 1-year-old child should receive prophylactic rifampin. 4 The 1-year-old-child should receive the Hib polysaccharide vaccine. 5 Only the infected child needs to be treated.

1. All family members, including the patient, should receive prophylactic rifampin

A 7-year-old boy presents with his mother with a 1-week history of wheezing and dyspnea on any exertion (with productive cough). On physical examination, bilateral rhonchi are heard. After a few days of treatment, spirometry is done on the patient. Total lung capacity (TLC) is 111% on spirometry. Pre-bronchodilators Post-bronchodilators FVC% 49 63 FEV1% 41 46 FEV1/FVC 49 55 Question What is the most likely diagnosis? 1 Asthma 2 Pneumonia 3 Pleural effusion 4 Kyphoscoliosis 5 Tuberculosis

1. Asthma

A 50-year-old man presents with multisystem failure secondary to bilateral pneumonia. He recently traveled from a work conference last week and presented to his primary care physician with fever, cough, and malaise 4 days ago. He was given a broad-spectrum antibiotic and he progressively became worse over the course of the antibiotic treatment. On exam, his body temperature is 40°C, respiration is 35/min, and pulse is 100/min. Laboratory examination is significant for impaired liver and renal function. Chest X-ray shows patchy infiltrates without evidence of consolidation. A sputum culture is performed and is significant for the presence of WBC in the Gram stain, but there are no organisms present. The culture result is negative. Question Based on your suspected diagnosis, what is the drug of choice in treating this patient? 1 Azithromycin 2 Metronidazole 3 Penicillin 4 Vancomycin 5 Cefazolin

1. Azithromycin

A 72-year-old man presents with an 8-month history of progressive exertional dyspnea accompanied by a dry and persistent hacking cough. He now feels slightly dyspneic at rest. He denies fever, chills, palpitations, chest pain, or peripheral edema. He denies any recent travel history, exposure to chemicals, and is a lifelong non-smoker. He is retired but previously worked as a school principal and teacher. His physical exam is remarkable for digital cyanosis and clubbing. Normal S1 and S2 without murmurs or gallops. Normal peripheral pulses bilaterally. A chest X-ray noted small lung volumes. Pulmonary function testing shows reduced TLC, FEV1, and normal FEV1/FVC ratio. Question What finding would you expect on a pulmonary exam? 1 Bibasilar dry inspiratory crackles 2 Dullness to percussion bilaterally 3 Expiratory wheezing 4 Hyperresonance to percussion 5 Inspiratory and expiratory wheezing

1. Bibasilar dry inspiratory crackles

A 5-week-old male infant presents with a 2-week history of prominent cough, nasal congestion, and wheezing. His symptoms have been getting progressively worse. Yesterday, the patient's mother took her son to her primary care doctor. The doctor started albuterol nebulizers every 4 hours and told her that the child had a viral infection and would get better. His mother is now particularly concerned that her child has had dry diapers for over 15 hours. The child has been irritable during this time. He has not had any fevers, vomiting, or diarrhea. The child is not tolerating breastfeeding or bottle-feeding well. The physical exam shows that the child is acyanotic and alert. The temperature is 97.8°F (36.5°C), respirations are 40/minute, and the pulse is 119/minute. There are no lymph nodes observed, and his tympanic membranes appear normal. There are rales noted diffusely on auscultation. The mucous membranes are moist and the skin has good turgor. You also detect conjunctivitis. Blood work is obtained; including a set of blood cultures, and a urine culture. The following lab values return: Hemoglobin 12.6 mg/dL Hematocrit 37.1% Platelet count 204 x 109 Eosinophilia count 6% The chest film demonstrates interstitial infiltrates and hyperinflation. Question What organism is causing the infant's symptoms? 1 Chlamydia trachomatis 2 Human parvovirus 3 Parainfluenza virus 4 Respiratory syncytial virus 5 Staphylococcal species

1. Chlamydia trachomatis

A 67-year-old Caucasian woman presents with shortness of breath and chest pain. She admits to getting home yesterday from a 2-day car ride after visiting her grandchildren. Her heart rate is 110 beats/minute and respirations are 22 breaths/minute. Blood pressure is 125/85 mm Hg. Her oxygen saturation is 99% on room air and temperature is 98.9° Fahrenheit. With additional information provided, her Wells Criteria Score is calculated to be 5. Question Which of the following is the most appropriate initial diagnostic test? 1 D-dimer 2 Echocardiogram 3 CT chest without contrast 4 ECG 5 CT chest pulmonary angiography

1. D-dimer

A 32-year-old African American woman with no significant past medical history has been referred to a pulmonologist; she presents with a 2-month history of progressive dyspnea. She notes associated low-grade fever, malaise, joint pain, and swollen neck glands. She denies a history of travel, cigarette smoking, drug use, or sexually transmitted diseases (she has not been sexually active in the past year). All other reviews of systems are negative. Her physical exam reveals tender nodular formations on her anterior lower extremities, parotid enlargement, hepatosplenomegaly, and cervical lymphadenopathy. Her vital signs, heart, and lungs are unremarkable. Diagnostic testing reveals leukopenia, increased ESR, hypercalcemia, hypercalciuria, elevations of serum ACE levels, and bilateral hilar adenopathy with diffuse reticular infiltrates. ANCA, ANA, and rheumatoid factor tests are negative. Histological assessment confirms the presence of noncaseating granulomas. Question What is the most likely diagnosis? 1 Sarcoidosis 2 Tuberculosis 3 Wegener's granulomatosis 4 Pneumocystis jirovecii pneumonia 5 Idiopathic pulmonary fibrosis

1. Sarcoidosis

A 5-month-old infant presents in the winter months with wheezing, rapid respirations (>45 breaths/min), and chest retractions. The patient has a 2-day history of rhinorrhea and low-grade fever. Breath sounds are normal, and there is no cyanosis. Question What test can confirm the most likely diagnosis? 1 Chest X-ray 2 Immunofluorescence of nasal secretion 3 Gram stain of the sputum 4 Blood gas analysis 5 Complete blood count with differential

2. Immunofluorescence of nasal secretion

A 43-year-old man without any significant PMHx presents with acute onset of a productive cough, shortness of breath, pleuritic chest pain, and fever. His sputum is described as "thick, brown-colored, and mucoid" but without blood. He also notes associated fatigue and night sweats. He denies chills, changes in weight, a history of travel, sick or confined contacts, exposure to animals, cigarette smoking, otalgia, sore throat, swollen glands, abdominal pain, diarrhea, rashes, myalgias, and arthralgias. His physical exam is remarkable for fever, tachycardia, and tachypnea. The lung exam is noteworthy for right lower lung field increased tactile fremitus, dullness to percussion, inspiratory crackles, and bronchial breath sounds. Question What is the most likely etiologic agent responsible for this patient's presentation? 1 Streptococcus pneumoniae 2 Histoplasma capsulatum 3 Chlamydia psittaci 4 Pseudomonas aeruginosa 5 Mycobacterium tuberculosis

1. Streptococcus pneumoniae

A 40-year-old man with chronic alcoholism presents with a cough that produces of large amounts of fetid sputum. He developed the cough about 4 weeks ago, and it has gradually worsened over time. The sputum is now foul-smelling and copious. He has had high intermittent pyrexia for 4 days. His vitals are: T 102°F, PR 98\min, RR 24\min, BP 140\80 mm Hg. HEENT exam shows poor dentition with multiple missing teeth, mild erythema to the nasal muscosa with clear drainage. RRR without a murmur. On lung auscultation, there is pleural rub and diminished air entry on the right side. The chest X-ray shows a large, irregularly shaped, dense opacity on the right side with a fluid level. Question How should this patient's diagnosis be regarded? 1 Only anaerobic organisms are responsible for this condition. 2 Aspiration is the most common cause. 3 Sputum culture is the best investigation. 4 CT has no role as an investigation in this condition. 5 Treatment is with antibiotics for 3 days.

2. Aspiration is the most common cause

A 25-year-old man with no significant medical history presents due to sudden onset of shortness of breath associated with right-sided chest pain. The pain is worse with inspiration but is unaffected by position. He states he was grocery shopping when it started. He denies chest trauma. Patient admits to an upper respiratory infection earlier in the month that had resolved without incident. He smokes 1 pack of cigarettes per day. On examination, he is afebrile, BP is 138/80 mm Hg, pulse is 124, respiratory rate is 24, and pulse oximetry is 94% on room air with mild respiratory distress. Trachea is midline. He has increased resonance to percussion with no breath sounds on the right anterior apex; the other lung fields are clear to auscultation. Heart is tachycardic with normal S1 and S2; no murmur, rubs, or gallops are present. Question What is the imaging of choice to make the diagnosis? 1 Chest computed tomography (CT) 2 Chest radiograph 3 Chest ultrasound 4 Electrocardiogram (ECG) 5 Spiral chest computed tomography (CT)

2. Chest radiograph

A 69-year-old man presents with dyspnea on exertion (climbing stairs and walking short distances) that has slowly progressed over the last year. He has fatigue, palpitations, intermittent retrosternal chest pain, lower extremity swelling, dizziness, and "feeling faint." Associated symptoms occur upon exertion. He denies fever, chills, weight changes, cough, abdominal pain, early satiety, nausea, vomiting, diarrhea, changes in urine color/odor, flank pain, hematuria, or dysuria. No cigarette, alcohol, or drug use. Cardiac exam shows increased pulmonic component of the second heart sound (P2), wide inspiratory splitting of S2 over the cardiac apex, right-sided S3 and S4 gallops, left parasternal lift, loud diastolic murmur increasing with inspiration and diminishing with Valsalva maneuver, prominent "A" waves in jugular venous pulsations, and increased JVD. Enlarged liver with hepatojugular reflux, peripheral edema, and ascites. EKG reveals peaked P waves, rightward axis deviation, and prominent R waves in the early V leads. 1. Myocardial infarction 2 Cor pulmonale 3 Primary biliary cirrhosis 4 Left ventricular heart failure 5 Pulmonary embolism

2. Cor pulmonale

An 80-year-old man has a past medical history of chronic obstructive pulmonary disease, persistent asthma, and hypertension. He presents due to acute chest pain, dyspnea, and pleurisy that began suddenly 30 minutes ago. He denies fever, chills, hemoptysis, wheezing, diaphoresis, cough, or abdominal pain. His vital signs are notable for tachycardia and tachypnea. His thorax demonstrates unilateral decreased tactile fremitus, hyperresonance to percussion, and decreased to absent breath sounds. There is no jugular venous distension, cyanosis or accessory muscle usage. He is identified as a poor surgical candidate, and he has had previous episodes of similar manifestations in the past. Question What medication would be recommended to treat the underlying condition given the recommendation not to pursue surgery? 1 Warfarin 2 Doxycycline 3 Furosemide 4 Levofloxacin 5 Prednisone

2. Doxycycline

A 64-year-old man with hypertension, coronary artery disease, and poorly-controlled left ventricular congestive heart failure presents with a 3-day history of insidious chest pain. Pain is made worse when he takes a deep breath in and when he coughs. He denies any relation of pain to position, activity, or food intake. He denies fever, chills, palpitations, sputum production, wheezing, abdominal pain, nausea, vomiting, diarrhea, or peripheral edema. His physical exam reveals a widespread friction rub upon inspiration, absent lung fremitus, and reduced lung sounds over the thoracic cavity. Question What additional physical exam finding would be most likely expected in this patient? 1 Vesicular breath sounds 2 Dullness to percussion 3 Tracheal shift to the affected side 4 Chest wall tenderness 5 Increased anteroposterior diameter

2. Dullness to percussion

A 57-year-old Caucasian man presents with worsening shortness of breath. While obtaining his history, you uncover that he has noted increasing shortness of breath with minor exertional activity and a persistent but non-productive cough. The patient admits to being a former smoker with a 34 pack-year history, admitting to cessation at age 50. He denies any known caustic occupational exposures and states he worked in an office his whole life. He admits to an uncle having some kind of breathing issues, although he is unsure of a definite diagnosis. Patient denies weight loss, fever, or significant recent illness. Physical examination is pertinent for significant clubbing of the fingers, inspiratory squeaks auscultated during the pulmonary exam, and a right-sided gallop found during the cardiac exam. What is the most likely diagnosis? 1 Sarcoidosis 2 Idiopathic pulmonary fibrosis 3 Silicosis 4 Cryptogenic organizing pneumonia 5 Lung cancer

2. Idiopathic pulmonary fibrosis

A 41-year-old woman presents due to worsening symptoms. She was diagnosed with idiopathic pulmonary hypertension about 2 years prior to presentation; she is on home oxygen therapy. She has longstanding fatigue and dyspnea, but she is now experiencing profound dyspnea with exertion, swelling in her ankles, some discomfort in her right upper abdomen, and the inability to breathe well when lying down. She has always been thin, but her weight has increased by 10 pounds in the last month. She denies fever and chills. She recently had an electrocardiogram (ECG), but she has not seen a healthcare provider to discuss the results. The ECG report indicates peaked p waves, right axis deviation, and tall r wave in V1. Question What is the most appropriate intervention for her current condition? 1 Prescribe a calcium channel blocker, such as verapamil. 2 Prescribe a diuretic, such as furosemide. 3 Prescribe a fluoroquinolone, such as levofloxacin. 4 Prescribe a lipase inhibitor, such as orlistat. 5 Prescribe a thiazolidinedione, such as pioglitazone.

2. Prescribe a diuretic, such as furosemide.

A 68-year-old man with a history of cirrhosis presents due to weight gain, increased girth, and shortness of breath. He denies fever or chills, cough, melena, hematemesis, hemoptysis, and confusion. He takes furosemide on a daily basis, and he has not missed any doses. On physical exam, his blood pressure is 120/80; pulse is 78, and respiratory rate is 18. He is alert and cooperative. Heart exam: regular rate and rhythm without murmurs. Lung exam: decreased breath sounds on right side. Abdominal exam: positive fluid wave, no tenderness to palpation. There is no hepatosplenomegaly appreciated. Chest X-ray shows a right-sided pleural effusion; it is free-flowing, as is evidenced on a decubitus film. Question What is the etiology of his pleural effusion? 1 Altered permeability of pleural membrane 2 Reduced intravascular oncotic pressure 3 Increased hydrostatic pressure 4 Decreased lymphatic drainage 5 Reduced pressure in pleural space

2. Reduced intravascular oncotic pressure

A 39-year-old previously well Caucasian man presents to the emergency department with a 10-day history of fever >101°F and acute dyspnea with pleuritic chest pain. His past medical history is notable only for childhood asthma (no recurrences since age 12) and appendectomy. He has no known drug allergies. He denies taking prescribed medications on a regular basis. Social history is notable for use of IV drugs. Vital signs show temperature 100.8°F, pulse 108, respirations 24, blood pressure 98/60. O2 saturation is 90% on room air. Physical examination reveals mild crackles of the mid-lung fields bilaterally and a grade II/VI soft systolic murmur, loudest at the left lower sternal border. A spiral CT reveals evidence of multiple pulmonary emboli. He is admitted to the general medical floor of an acute care hospital. Additional diagnostic tests are ordered; preliminary results of blood cultures showed 4+ growth of gram-positive cocci. Infectious Diseases is consulted and he is started on an IV antibiotic regimen. Question What is the most likely source of the multiple pulmonary emboli in this patient? 1 Deep venous thrombosis 2 Tricuspid valve vegetation 3 Left atrial thrombosis from atrial fibrillation 4 Disseminated intravascular coagulation 5 Metastatic cancer

2. Tricuspid valve vegetation

A 37-year-old man with an unremarkable past medical history presents during a cold winter day with a 10-day history of acute onset of productive cough with a moderate amount of yellow sputum. There is associated fever, shortness of breath, and malaise. He denies recent travel, sick contacts, occupational exposure, and a history of smoking or alcohol use. He denies arthralgias, chills, wheezing, abdominal pain, nausea, vomiting, diarrhea, edema, or rashes. His physical exam is remarkable for fever, tachypnea, reduced fremitus, dullness to percussion, and basilar crackles in the right lower lung field. Question What additional presentation finding or set of findings would be most likely in this patient? 1 Chest pain precipitated by exertion and an S4 gallop 2 Unilateral sharp inspiratory chest pain and thoracic friction rub 3 Chest pain relieved by antacids and provoked by food intake 4 A vesicular rash on the thorax preceded by paresthesias 5 Hemoptysis, tachypnea, unilateral lower extremity edema

2. Unilateral sharp inspiratory chest pain and thoracic friction rub

A 4-year-old boy presents with a 3-day history of fever with chills, cough, and fast breathing. His parents report decreased oral intake and increased difficulty breathing with retractions over the last 24 hours. His initial vital signs are heart rate 144 bpm; respiratory rate 32/min; temperature 101.3°F (38.5°C); oxyhemoglobin saturation 89% on room air. He is immediately started on supplemental oxygen, and his oxyhemoglobin saturation improves to 95%. Subsequent evaluation, including a chest X-ray, is suggestive of right middle and lower lobe pneumonia. Question What is the mechanism for the low oxyhemoglobin saturation in this patient? 1 Hypoventilation 2 Ventilation-perfusion mismatch 3 Diffusion block 4 Shunt 5 Increased dead space in lungs

2. Ventilation-perfusion mismatch

A 15-year-old girl with a history of mild asthma has had worsening episodes of cough, wheezing, and increasing bloody sputum over the past 5 months. She denies weight loss, decreased appetite, lethargy, or travel. She has increased her bronchodilator use, but she had not sought further care. Her mother has noted facial flushing with sweating that sometimes appears when she feels stressed—brief at first but lasting longer now. On exam, her respiratory rate is 32 breaths/min, temperature is 98.6°F, heart rate 84 bpm, BP 114/76 mm Hg, oxygen saturation is 94%. Her throat is clear, RRR without murmur; on auscultation, breath sounds over the left hemithorax are diminished without retractions or wheezes; there are few fine crackles at the base. Right side is clear. Remainder of the exam is normal. Chest X-ray reveals a round area of increased opacification near the right hilar region. CBC shows normal white count and differential. Question What diagnosis would most easily explain the patient's symptoms? 1 Pulmonary embolism 2 Bacterial pneumonia 3 Bronchial carcinoid tumor 4 Pulmonary hemosiderosis 5 Vascular malformation

3. Bronchial carcinoid tumor

A 55-year-old woman presents with a slight cough she has had for about a week. She is a nonsmoker, and she does not remember having a fever or feeling sick. The patient currently works as a third-grade teacher and has done this for 30 years. She has no past medical history of significant pulmonary diagnoses. Auscultation of the chest reveals clear lung fields. A chest X-ray shows a subpleural "coin lesion" in the right upper lobe. What is the most likely diagnosis? 1 Small cell anaplastic carcinoma 2 Bronchiectasis 3 Silicosis 4 Granuloma 5 Exogenous lipid pneumonia

4. Granuloma

A 64-year-old man presents with a 3-day history of insidious chest pain. He has a past medical history of hypertension, coronary artery disease, and poorly controlled left ventricular congestive heart failure due to medication noncompliance. Pain is made worse when he takes a deep breath and when he coughs. He denies any relation of pain to position, activity, or food intake. He denies fever, chills, palpitations, sputum production, wheezing, shortness of breath, abdominal pain, nausea, vomiting, diarrhea, and peripheral edema. He denies any relation of pain to position, activity, or food intake and is able to perform his normal daily activities. His physical exam reveals no respiratory distress, cyanosis, or accessory muscle usage. There are bibasilar thoracic friction rubs upon inspiration, an absence of lung fremitus, dullness to percussion, and reduced lung sounds. A chest X-ray is performed. Refer to the image. Question What health maintenance approach should be recommended at this time? 1 Aggressive restriction of dietary fat intake 2 Explaining the necessity of an emergent and repeated thoracentesis 3 Counseling regarding compliance with heart failure medications 4 Prophylactic use of broad-spectrum antibiotics 5 No need to reevaluate onset of shortness of breath in this patient's chronic condition

3. Counseling regarding compliance with heart failure medications

A 14-year-old boy presents with worsening shortness of breath; it most often occurs when he plays soccer. He often awakens in the middle of the night due to "attacks": he starts to feel anxious because he feels like he cannot breathe, and he experiences chest tightness. He suffers from a dry cough, especially after playing sports. His mother has put a humidifier in his room and has him use his sister's inhaler, which seems to help temporarily; the boy has been using it 5-6 times daily. The family history is significant for asthma in his sister, father, and three other paternal relatives. The patient and his mother are not aware of any allergies. He denies fever, chills, and chest pain. In between "attacks," he feels well and normal. The patient's past medical history is noncontributory. There are no known medical conditions; he has no drug allergies, and he has not had any surgeries. Other than the aforementioned inhaler, he does not take any medications. Question In addition to his own albuterol inhaler, what medication should be prescribed for this patient? 1 Burst of oral prednisone 2 Daily inhaled salmeterol 3 Daily low-dose inhaled budesonide 4 Daily oral zileuton 5 Subcutaneous injections of omalizumab

3. Daily low-dose inhaled budesonide

A 58-year-old woman with a past medical history of hypertension, hyperlipidemia, breast cancer, hip fractures, and coronary artery disease is being evaluated for acute-onset severe left-sided pleuritic chest pain over the course of the last 2 hours. The pain is associated with feelings of anxiety, hemoptysis, shortness of breath, and nausea. She "feels warm" but denies chills, palpitations, wheezing, edema, vomiting, abdominal pain, abnormal bowel habits, or dietary intolerances. She admits to a 30 pack-year smoking history but denies drug or alcohol use. Upon physical exam, she is found to be febrile, hypotensive, tachycardic, tachypneic, diaphoretic, and in acute painful distress. There are perioral cyanosis and a pleural friction rub to the left lung fields; the remainder of the exam is normal. Question What is the most appropriate therapeutic intervention for this patient at this time? 1 Indomethacin 2 Doxycycline 3 Heparin 4 Prednisolone 5 Albuterol

3. Heparin

A 53-year-old man with a 40 pack-year smoking history presents with a 10-month history of an intermittent cough with productive sputum. He admits to progressive exertional shortness of breath, which recently has limited his activity to climbing 1 flight of stairs or walking 3 city blocks. He denies diaphoresis, fever, chills, chest pain, palpitations, audible wheezing, pleurisy, peripheral edema, hemoptysis, abdominal pain, reflux, regurgitation, diarrhea, melena, or hematochezia. He also denies travel, sick contacts, and drug or alcohol use. His general survey reveals an overweight male with an odor of smoke and nicotine staining of his fingernails. His nails also demonstrate digital clubbing. His pulmonary exam reveals a prolonged expiratory phase, barrel chest, poor diaphragmatic excursion, and wheezing to auscultation. Pulmonary function testing shows airflow obstruction with a reduction in FEV1 and FEV1/FVC ratio; increases in total lung capacity, functional residual capacity, and residual volume were noted. Question What intervention has been demonstrated to influence the natural history of this patient's illness? 1 Intravenous diuretics 2 Inhaled anticholinergic agents 3 Smoking cessation 4 Long-acting inhaled agonists and glucocorticoids 5 Antibiotic prophylaxis

3. Smoking cessation

A 3-year-old boy presents due to severe bouts of coughing for the last 10 days that started as a common cold. During coughing, his face becomes red. The episode of cough often ends with a loud sound during breathing or vomiting. He also has low-grade fever off and on for the last 10 days. Immunization records of the child are not available. Blood count shows leukocytosis with lymphocytosis. Question What is the antibiotic of choice to treat and prevent spread of the infection? 1 Cotrimoxazole 2 Erythromycin 3 Rifampicin 4 Azithromycin 5 Ampicillin

4. Azithromycin

You are evaluating a 78-year-old man who lives in a nursing home due to moderate Parkinsonism. The patient does not have any specific complaints, but nursing home staff have noticed that he does not seem as alert as usual and is spending more time in his room sleeping. He is also eating less and has dropped 2 lb in the last 2 weeks. The patient's temp is 100.8°F; pulse 100 bpm; respiration 25/min; and bp 120/70 mm Hg. On exam, he is aware of the place, but not the exact time or date. His HEENT exam is unremarkable. His cardiac exam reveals tachycardia, but no murmurs or rubs. His lung sounds are decreased at the bases. Question Based on this presentation, what should be included as part of the initial workup? 1 Psychiatric evaluation 2 CT of the head 3 Nutritional evaluation 4 Complete blood count 5 Erythrocyte sedimentation rate (ESR)

4. Complete blood count

A 55-year-old woman presents with constant fatigue. She has experienced multiple episodes of falling asleep at work and dozing off while waiting at red lights. Her husband consistently complains about her snoring and snorting while asleep; it wakes him at night. On physical exam, she is 5'2" and 205 lb. Her blood pressure is 150/90 mm Hg, her pulse is 82 BPM, her respirations are 16/min, and she is afebrile. The rest of her physical exam is unremarkable. Laboratory studies, including thyroid function tests, are within normal limits. After advising her on diet and weight loss, you schedule her for an overnight sleep study. Question What results do you expect the study to show? 1 Cheyne-Stokes breathing pattern 2 Episodes of absent airflow with absent respiratory effort 3 No change in oxygen saturation on pulse oximetry 4 Five or more episodes of apnea or hypopnea per hour 5 Breathing pattern associated with atrial fibrillation

4. Five or more episodes of apnea or hypopnea per hour

A 12-year-old boy with cystic fibrosis (CF) presents for a periodic evaluation visit. Weight gain has been stable, but he has had three pulmonary exacerbations in the past year. His participation in physical activity has decreased during that time because his parents believe that exercise will be detrimental to him in his weakened condition. He has many friends who participate in athletics and physical activity, however, and he would like to join them. Past medical history is otherwise unremarkable except for occasional episodes of sinusitis. Vital signs reveal a respiratory rate of 20 at rest, with scattered crackles and wheezes at both lung bases. Pulse oximetry is 93% at rest. Question What advice would you give? 1 Cystic fibrosis is a contraindication to competitive sports. 2 Exercise training is of little benefit in cystic fibrosis. 3 Activity should be minimal in patients with frequent exacerbations. 4 Long-term regular exercise training should be initiated. 5 Parents should remain neutral on the issue of exercise.

4. Long-term regular exercise training should be initiated

A 72-year-old man presents with longstanding and increasing dyspnea and a 3-day history of shortness of breath, coughing, and unilateral sharp chest pain. Pain is worse when he takes a deep inspiration and when he coughs. PMH is significant for a smoking history of 2 packs/day for 40 years (80 pack-years). The patient has other co-morbidities, including CAD, COPD, CHF, hypertension, and dyslipidemia. He denies recent travel, sick contacts, occupational exposure, and drug or alcohol use. Vital signs are BP 150/90, HR 96, RR 26, O2 94% on room air. Chest examination of the left posterior chest reveals a dull percussion note, inaudible bronchovesicular breath sounds, decreased tactile fremitus, a pleural friction rub on inspiration and diminished voice sounds below the sixth intercostal space. Significant bilateral lower extremity edema is also noted. A chest X-ray reveals blunting of the costophrenic angle. Question What is the most likely diagnosis? 1 Pneumothorax 2 Pulmonary thromboembolism 3 Pulmonary fibrosis 4 Pleural effusion 5 Bronchiectasis

4. Pleural Effusion

A 69-year-old woman presents with shortness of breath. She states it has been worsening over the last 3-4 days; she also is experiencing increased fatigue. The patient is not on any daily medications other than over-the-counter multivitamins, and she has no pertinent past medical history. Physical examination is significant for an oral temperature of 101.5°F, and during auscultation, there are absent breath sounds noted in the right lower lung field. Tactile fremitus reveals an absent result in that same lung field, and percussion over that area creates a dull percussion note. Question What is the most likely diagnosis? 1 Asthma 2 COPD 3 Early left-sided heart failure 4 Pleural effusion 5 Pneumothorax

4. Pleural effusion

A 22-year-old man presents with sudden onset of shortness of breath and right-sided chest pain. Symptoms began abruptly yesterday. He felt well prior to the onset of symptoms. He denies fever, hemoptysis, and upper respiratory symptoms. He smokes one pack per day; he has an otherwise non-contributory past medical history. On physical exam, the patient is in mild respiratory distress. He has a slightly elevated heart rate and respiratory rate. He is normotensive. His trachea appears deviated to the left. On pulmonary exam, breath sounds are diminished on the right. Hyperresonance is noted on percussion of the right chest compared to the left. Other than tachycardia, his cardiovascular exam is normal. Question What test finding is most diagnostic for your suspected diagnosis of this patient? 1 Blunting of costophrenic angles on chest X-ray (CXR) 2 Increased pH on arterial blood gas (ABG) 3 Oxygen saturation less than 90% on pulse oximetry 4 Pleural line on chest X-ray (CXR) 5 Sputum smear positive for acid-fast bacilli (AFB)

4. Pleural line on chest X-ray

A 50-year-old man with no significant past medical history presents for a follow-up. He notes that he has developed increased non-productive cough and shortness of breath over the past week. He has had a chronically progressive cough and shortness of breath upon exertion over the past year. His occupational history reveals coal mining for 25 years. He denies any smoking, alcohol consumption, travel history, or sick contacts. He also denies fever, chills, sore throat, otalgia, chest or abdominal pain, peripheral edema, rashes, or pruritus. A chest radiograph was performed. Refer to the image. Question What treatment would be most beneficial? 1 Trexall 2 Oseltamivir 3 Amoxicillin-clavulanate 4 Prednisone 5. Acyclovir

4. Prednisone

A 70-year-old man with type 2 diabetes mellitus, hyperlipidemia, homocysteinemia, and metabolic syndrome presents with a 5-month history of excessive daytime sleepiness, a lack of refreshing sleep, a depressed mood, and an inability to focus at work and while driving. Additionally, he has been told by his wife that he snores rather loudly while sleeping. He denies fever, chills, headache, cold intolerance, weight loss, hair changes, hoarseness, dysphagia, chest pain, edema, palpitations, or changes in his bowel habits. On physical exam, he is found to be hypertensive. He has elevated BMI with abdominal obesity, and he has an enlarged neck circumference; no other abnormalities are noted. Question What pathological mechanism best accounts for this patient's presentation? 1 Bacterial overgrowth of Streptococcus, causing pharyngeal inflammation 2 Lymphocytic infiltration of the thyroid, causing fibrosis of the thyroid follicles 3 Compression of the superior vena cava by an infiltrating mediastinal neoplasm 4 Reduced inspiratory patency of the airway due to relaxation of the muscles 5 Lower airway inflammation with mucosal thickening and mucus hypersecretion

4. Reduced inspiratory patency of the airway due to relaxation of the muscles

An 18-month-old boy presents with a 2-day history of runny nose, slight cough, and low-grade fever. Over the last 24 hours, however, his condition has worsened; the child is tachypneic on exam. He is wheezing, and his breaths are associated with nasal flaring and chest retraction. Question What is the most common etiology of this patient's condition? 1 Adenovirus 2 Haemophilus influenzae 3 Influenza virus 4 Respiratory syncytial virus 5 Streptococcus pneumoniae

4. Respiratory syncytial virus

A 62-year-old African American woman presents with a persistent cough and shortness of breath. Bronchoscopy is performed and the report includes the following description: "2 x 2 cm non-necrotizing granuloma in the left upper lung field and a 1 x 1 cm non-necrotizing granuloma in the right middle lung field." Question What is the most likely diagnosis? 1 Hypersensitivity pneumonitis 2 Lung cancer 3 Mononucleosis 4 Sarcoidosis 5 Sickle cell disease

4. Sarcoidosis

A 68-year-old non-smoking man presents to your pulmonology practice for long-standing dyspnea and non-productive cough. The patient has had the cough and progressively worsening dyspnea for about 1.5 years with no current exacerbation. He denies fevers, chills, night sweats, and any other symptoms. He denies unusual travel, hobbies, or occupational exposures. He does, however, endorse relatively frequent reflux symptoms. The rest of his review of systems is negative. He has had a workup through his family practice and was then sent to the cardiologist, who ruled out cardiovascular causes of his dyspnea. He has been given trials of various antibiotics, inhalers, and steroids, all without improvement in symptoms, despite good compliance. He currently takes no medications. Several tests have been performed, and results are shown in the table. Blood count, metabolic panel, HIV, and autoimmune markers Normal High-resolution computed tomography (HSCT) scan Few reticular opacities Pulmonary function test (PFT) Restrictive impairment and reduced perfusion of carbon monoxide Physical exam is significant for fine inspiratory bibasilar crackles and clubbing in the fingers. An occasional dry cough is noted. Question What intervention is most likely to prevent complications in this patient? 1 Cisplatin-based chemotherapy 2 Inhaled beta agonist 3 Inhaled corticosteroid 4 Treatment of gastroesophageal reflux disease 5 Avoidance of supplemental oxygen

4. Treatment of GERD

A 3-year-old girl presents with a 2-day history of a sore throat and fever. This morning, she was hoarse and seemed to be having more difficulty breathing. On exam, she appears to be in distress and has an oral temperature of 100.0°F. Tympanic membranes are pink but not bulging. Nares are patent without rhinorrhea. She has a barking cough, stridor at rest, and nasal flaring. Question What treatment is most appropriate in the care of this child? 1 Admit patient, start specific antiviral therapy and hydration. 2 Discharge patient home on a broad spectrum antibiotic for 14 days. 3 Discharge patient home after administering parainfluenza vaccine. 4 Discharge home and advise parents to use cool mist vaporizer. 5 Admit patient, start humidified oxygen and racemic epinephrine.

5. Admit patient, start humidified oxygen and racemic epinephrine

An 84-year-old woman presents with a 6-hour history of dyspnea, non-productive cough, hemoptysis, and a "sharp, stabbing" pleuritic chest pain. Her review of systems is negative for any fever, chills, palpitations, wheezing, abdominal pain, nausea, vomiting, diarrhea, and rashes. She has a past medical history of myocardial infarction, congestive heart failure, dyslipidemia, asthma, and lung cancer. Social history is significant for a 50 pack-year smoking history, but she quit 10 years ago. She denies any alcohol or recreational drug use, sick contacts, or recent travel. She also denies any recent hospitalizations or surgeries. On physical exam, skin is warm and dry without rashes. There is abdominal distension, hepatosplenomegaly, supraclavicular lymphadenopathy, and 2+ lower extremity pitting edema (right greater than left) noted. The cardiac exam reveals tachycardia, jugular venous distension of 6 cm, and an S3 gallop. Pulmonary exam reveals tachypnea, diffuse dullness to percussion, decreased tactile fremitus, and absent breath sounds. She undergoes diagnostic thoracentesis. Laboratory findings of pleural fluid are listed below. Appearance Clear Pleural fluid LDH 160 units/liter Pleural-to-serum protein 0.2 Pleural-to-serum LDH 0.3 Pleural fluid glucose Negative Pleural fluid WBCs Negative Pleural fluid pH Within normal limits Pleural fluid RBCs Negative Question What is the most likely contributory etiology? 1 Pulmonary embolization 2 Lung cancer 3 Bacterial pneumonia 4 Cirrhosis 5 Congestive heart failure

5. Congestive Heart Failure

A 40-year-old man with no significant past medical history presents with a 2-day history of alternating fever and rigors, diaphoresis, fatigue, and a productive cough. He admits to mucoid sputum of moderate quantities. He denies a history of smoking, alcohol use, recent travel, or sick contacts. He further denies chest pain, palpitations, hemoptysis, rashes, abdominal pain, nausea, vomiting, or diarrhea. On physical exam, he is found to be tachypneic and was observed to be intermittently coughing. The pulmonary exam was notable for bronchial breath sounds over the right anterior fourth, fifth, and sixth intercostal spaces. A chest radiograph revealed a right middle lobe consolidation. Question What additional physical exam finding would be consistent with this patient's most likely diagnosis? 1 Cheyne-Stokes respiration 2 Bradycardia 3 Decreased tactile fremitus 4 Decreased whispered pectoriloquy 5 Dullness to percussion

5. Dullness to percussion

A 17-year-old boy presents with intermittent bouts of shortness of breath, coughing, and chest tightness. The symptoms most often occur during football practice, sometimes when he is just standing outdoors in cold weather. He denies palpitations, fever, and chills. Past medical history is non-contributory. He is a non-smoker. Physical exam is unremarkable. Vital signs are as follows: BMI 19 kg/m2, BP 116/70 mm Hg, HR 80 bpm, SpO2 99% on room air. His physician orders pulmonary function testing (PFT) and a chest X-ray (CXR). Question Based on the most likely diagnosis, what chest X-ray finding is most likely? 1 Hyperinflation 2 Ground glass shadowing 3 Bronchial wall thickening 4 Perihilar fluffy infiltrates 5 Normal chest X-ray

5. Normal chest X-ray

A 56-year-old man presents for a routine physical exam. His medical history is significant for hypertension and non-insulin-dependent diabetes for which he takes hydrochlorothiazide and metformin, respectively. His body mass index is 33 kg/m2. He states that he knows he should exercise more, but he lacks energy. Despite getting 7-8 hours of sleep every night, he wakes up feeling unrefreshed. In addition, his wife says he snores so loudly that sometimes she asks him to sleep on the couch. He deliberately limits his caffeine intake to 2 cups of coffee in the morning. He rarely drinks alcohol, and he denies non-prescription drug use. Question Based on the patient's presentation, what is the most likely diagnosis? 1 Central sleep apnea 2 Insomnia 3 Medication side effect 4 Narcolepsy 5 Obstructive sleep apnea

5. Obstructive sleep apnea

A 22-year-old man presents with a sudden onset of shortness of breath and right-sided chest pain. Symptoms began yesterday, and he felt well prior to the onset of symptoms. He denies fever, hemoptysis, and upper respiratory symptoms. He is a 1 pack-per-day smoker; otherwise, he has a noncontributory past medical history. On physical exam, the patient is in mild respiratory distress, with a slightly elevated heart rate and respiratory rate. He is normotensive. His trachea appears deviated to the left. On pulmonary exam, breath sounds are diminished on the right. Hyperresonance is noted on percussion of the right chest compared to the left. Other than tachycardia, his cardiovascular exam is normal. A chest X-ray is obtained, and a pleural line is visible. Question What is the most likely diagnosis? 1 Bronchiectasis 2 Bronchitis 3 Foreign body in the bronchus 4 Pneumonia 5 Pneumothorax

5. Pneumothorax

A 6-week-old male infant presents with a 4-day history of cough and nasal congestion. He occasionally has a bluish tint around his lips while sleeping. No history of fever. Older siblings have upper respiratory infections. Appetite has been decreased due to the copious nasal secretions, but he has a normal urine output. He was delivered at 34 weeks. He had mild respiratory distress syndrome—2 days on a ventilator in the NICU. He went home in 10 days and has done well since. No immunizations. Physical exam reveals mild respiratory distress, respirations 52/min, with slight intercostal retractions. Temperature is 100.2°F, HR 130/min. Perioral duskiness is seen. Oxygen saturation at room air is 83%. HEENT exam otherwise normal. Chest exam shows coarse rhonchi, expiratory wheezes. Heart rate and rhythm regular. No murmurs appreciated. Abdomen is soft and non-tender. Neurological is intact. Chest X-ray shows mild hyperexpansion, no consolidation. Nasal swab for respiratory syncytial virus is positive. He is admitted to the hospital. Question In addition to supportive therapy, what is the most appropriate treatment? 1 Aerosolized albuterol 2 Intravenous steroids 3 Empiric antibiotics 4 Aerosolized ribavirin 5 Supplemental oxygen

5. Supplemental Oxygen

A 47-year-old previously healthy Caucasian woman presents due to a 4-week history of dyspnea that started approximately when on vacation in Colorado. She initially attributed these symptoms to the altitude, but she continued to have shortness of breath with mild activity: walking >100 feet, walking up a flight of stairs, housekeeping. Her symptoms resolve with rest. She also reports mild exertional chest tightness and easy fatigability. She denies paroxysmal nocturnal dyspnea, orthopnea, edema, palpitations, and syncope. Past medical history includes usual childhood illnesses. She has no previous surgeries and no known allergies. Medications include a daily multivitamin and occasional Tylenol for headaches. She is a non-smoker, rarely drinks, and denies recreational drugs. Physical exam reveals an alert white woman in no acute distress: temp 98.2°F, pulse 80 and regular, respirations 16, BP 136/82, O2 sat 96% on room air. There is no obvious jugular venous distention. Respirations are non-labored. Lung fields are clear to auscultation and percussion. No rhonchi, rales, or wheezes are present. Heart shows RV heave present, normal S1 with fixed, split S2 with prominent P2 component. Grade II/VI systolic murmur is present at the left upper sternal border at the second intercostal space. The remainder of a complete physical examination is within normal limits. CBC and BMP are unremarkable. Free T4 and TSH are within normal limits. EKG shows normal sinus rhythm with right ventricular hypertrophy, right atrial enlargement, and right axis deviation. There is an RSR in leads v1 and v2. Question What initial diagnostic test should you order to confirm the suspected diagnosis? 1 Exercise stress test 2 Holter monitor 3 Nuclear stress test 4 Right and left heart catheterization 5 Transthoracic echocardiogram

5. Transthoracic echocardiogram

A 72-year-old man presents due to worsening shortness of breath, orthopnea, and chest pain; symptoms have been occurring for the past few weeks. The patient admits to some chronic heart problems, as well as fatigue, dyspnea, and a non-productive cough. He feels like symptoms have worsened recently. He denies fever, chills, and a productive cough. On physical exam, the man has mildly increased respiratory effort, but he does not appear in distress. He is barrel-chested. His breath sounds are diminished bilaterally, with dullness to percussion over the right and left lower lungs. No pleural friction rub is noted. On cardiovascular exam, an S3 gallop and mild tachycardia (110 bpm) are noted. Clubbing of the fingers, dependent edema in the lower extremities, and jugular venous distention are also noted. His cardiac enzymes and electrocardiogram demonstrate no acute cardiac pathology. Pleural fluid and cardiomegaly are found on the chest X-ray, and a thoracentesis is performed. The pleural fluid is generally clear in color, testing negative for chylomicrons and triglycerides. It has low levels of red blood cells, white blood cells, protein, and lactate dehydrogenase (LDH). Question What is the likely underlying mechanism for the pleural effusion in this patient? 1 Chylothorax from disruption of the thoracic duct 2 Empyema from infection in the pleural space 3 Exudates from local inflammation in capillary beds 4 Hemothorax 5 Transudates from increased hydrostatic pressure or decreased oncotic pressure

5. Transudates from increased hydrostatic pressure or decreased oncotic pressure


Kaugnay na mga set ng pag-aaral

Physics - Chapter 8: Rotational Motion

View Set

PHARM - Neuro practice questions

View Set

Psych Chapter 7 Online Quiz and Learning Curve

View Set

Integrated Chinese Lesson 6 Dialoague 2

View Set

Nurs 300 - Potassium Balance and Imbalances

View Set

Model 4 Prin. Bus. Fin. " Your Business Snapshot"

View Set

DNA Replication, Genetic Code, Transcription Quiz

View Set